.

b) Lấy được ít nhất một quả cầu trắng

Answers

Answer 1

Answer:

I don't know the answer. I don't know the answer. I don't know the answer. I don't know the answer. I don't know the answer. I don't know the answer. I don't know the answer. I don't know the answer. I don't know the answer. I don't know the answer. I don't know the answer. I don't know the answer. I don't know the answer. I don't know the answer. I don't know the answer. I don't know the answer. I don't know the answer. I don't know the answer. I don't know the answer. I don't know the answer.


Related Questions

1+9+40-40+40+10x2 equals what?

Answers

Answer:

70

Step-by-step explanation:

1+9+40-40+40+10*2

----------------------- 20

10+40-40+40+20

50 (-40+40 cancels out to 0) +20

50+20= 70

(Following PEMDAS)

Multiply, add, subtract.

if you were to instead go through it left to right, you would get 120, which is incorrect.

Write the quadratic function f(x) = x2 - 2x - 8 in factored form.
A) f(x) =(x - 4)(x - 2)
B) f(x) =(x + 4)(x - 2)
C) f(x) =(x - 4)(x + 2)
D) Rx) =(x + 4)(x + 2)

Answers

Answer:

Hello,

answer C

Step-by-step explanation:

[tex]f(x)=x^2+2x-8\\\\=x^2-4x+2x-8\\\\=x(x-4)+2(x-4)\\\\=(x-4)(x+2)\\\\Answer\ C[/tex]

A certain positive integer has exactly 20 positive divisors.

What is the largest number of primes that could divide the integer?

thx for the help in advance

Answers

9514 1404 393

Answer:

  3

Step-by-step explanation:

20 has at most 3 proper factors greater than 1: 2×2×5. Each of these can represent a prime factor of the number of interest, and is 1 more than that prime's power. That is, the number of interest (n) will have at most 3 prime factors p, q, r, and will be ...

  n = p·q·r^4

_____

For some prime factorization ...

  [tex]\displaystyle n=\prod_{k=1}^m{p_k^{q_k}}[/tex]

The total number of divisors of n is ...

  [tex]\displaystyle\prod_{k=1}^m{(q_k+1)}[/tex]

Find the minimum turning point of y = x^2 + x - 12

Answers

Answer:

(x+4)(x-3)

Step-by-step explanation:

x^2+x-12

=x^2+(4-3)x-12

=x^2+4x-3x-12

=x (x+4)-3 (x+4)

=(x+4)(x-3)

Answer:x=6

Step-by-step explanation:

Maximum and minimum value

Answers

Answer:

Maximum: 1

Minimum: -1

Step-by-step explanation:

To solve this, we can find the derivative of the function and then solve for when the derivative is equal to 0, finding the local maximum and minimum values in this function. Then, we can compare those points to the endpoints of the interval to find the maximum and minimum.

Using the Quotient Rule, we can say that

(f/g) ' = (f'g - fg')/(g²)

[tex]\frac{2x}{x^2+1}dx = \frac{\frac{d}{dx}(2x) * (x^2+1) - \frac{d}{dx} (x^2+1) * (2x) }{(x^2+1)^2} \\= \frac{2 *(x^2+1) - (2x)*(2x)}{(x^2+1)^2} \\[/tex]

Since we just need to solve for 0, we can disregard for the bottom part, resulting in

2(x²+1)-(2x)(2x) = 0

2x²+2-4x² = 0

2-2x² = 0

add 2x² to both sides to isolate the x² and its coefficient

2x² = 2

divide both sides by 2

x²=1

x = ±1

Therefore, we have 4 possible values for the maximum and minimum over this interval:

-2, -1, 0, 1

Trying each of these numbers into f(x),

f(-2) = -4/5

f(-1) = -2/2 = -1

f(0) = 0/1 = 0

f(1) = 2/2 = 1

Therefore, the maximum and minimum values of the function over the given interval are 1 and -1 respectively

Please help me please! I really need it! Thank you so much!!!!!!!!!!! Sorry Quality is really bad

Answers

Answer:

 7[tex]\frac{5}{6}[/tex]

Step-by-step explanation:

- 2[tex]\frac{1}{3}[/tex] - ( - 10[tex]\frac{1}{6}[/tex] ) = - 2[tex]\frac{1}{3}[/tex] + 10[tex]\frac{1}{6}[/tex] = - 2[tex]\frac{2}{6}[/tex] + 10[tex]\frac{1}{6}[/tex] = ( 10 - 2 ) + ( [tex]\frac{1}{6}[/tex] - [tex]\frac{2}{6}[/tex] ) = 8 - [tex]\frac{1}{6}[/tex] = 7 + ( [tex]\frac{6}{6}[/tex] - [tex]\frac{1}{6}[/tex] ) = 7[tex]\frac{5}{6}[/tex]

Help please! it’s due soon

Answers

Answer:

33. That means (A) can be any number and b times any number wilo get 0 so any numbr will do

34.It is wrong because if x equals 40 anything multiply that will be more than that and so isnt a good reasoning

Which expression is equivalent to the following complex fraction?

Answers

Answer:

The correct choice is option B. y+1/y-1

Step-by-step explanation:

(a) If y varies directly as x, and y=24 when x=16, find y when x=12.

(b) In a formula, z varies inversely as P. If Z is 200 when P is 4, find z when P is 10.

Answers

Answer:

(a) 18

(b) 80

Step-by-step explanation:

(a) let y=kx

so, 24=16k

or, k=24/16 or, k=3/2

now, putting x=12

y=kx

or, y=(3/2)×12

or, y=3×12/2

or, y=36/2

or, y=18

(b) let P=k/z

so, 4=k/200

or, k=800

when P = 10

P=k/z

or, 10=800/z

or, z=800/10

or, z=80

What is the range of the function in the graph?

Answers

Answer:

b=70

Step-by-step explanation:

The range is the value that the output takes

In this graph, the output is constant at 70

The output value is b

b=70

r-(-4x - (y + y)); use x = -4, and y = -3 evaluate ​

Answers

Answer:

insert values of x and y

r - ( -4(-4) - ( -3-3))

r - ( 16 +6 )

r - 22

Doneeeeeeeeeeeeeeeeee3

Find the measure of angle x in the figure below:

A triangle is shown. At the top vertex of the triangle is a horizontal line aligned to the base of the triangle. The angle formed between the horizontal line and the left edge of the triangle is shown as 57 degrees, the angle formed between the horizontal line and the right edge of the triangle is shown as 61 degrees. The angle at the top vertex of the triangle is labeled as y, and the interior angle on the right is labeled as 67 degrees. The interior angle on the left is labeled as x.

35°
47°
51°
62°

Answers

I think is 62 but I’m not sure

Your answer iss...

It is 51º

Solve 7 ( x + 1 ) + 2 = 5x + 15

Answers

Answer:

x = 3

Step-by-step explanation:

7(x + 1) + 2 = 5x + 15

~Simplify left side

7x + 7 + 2 = 5x + 15

~Combine like terms

7x + 9 = 5x + 15

~Subtract 9 to both sides

7x = 5x + 6

~Subtract 5x to both sides

2x = 6

~Divide 2 to both sides

x = 3

Best of Luck!

A ball is thrown straight up into the air from an initial height of 5 feet at time t = 0. The height, in feet, of the ball above the ground is given by h(t), where t is measured in seconds for 0 ≤ t ≤ 15. Based on the values of t and h(t) given in the table, for which value of t would the speed of the ball most likely be the greatest?

t (seconds) 0 3 6 9 12 15
h(t) (feet) 5 12 15 11 6 0

Select one:
a. 2 seconds
b. 5 seconds
c. 9 seconds
d. 15 seconds

Answers

Answer:

5 seconds

Step-by-step explanation:

The speed of an object is the rate of distance over time. The value of time (t) at the greatest speed of the ball is at 2 seconds

First, we calculate the speed using:

[tex]Speed = \frac{h(t)}{t}[/tex] --- i.e. distance/time

At t = 0, h(t) = 5

So;

[tex]Speed = \frac{5}{0} = unde fine d[/tex]

At t = 3, h(t) = 12

[tex]Speed = \frac{12}{3} = 4[/tex]

At t = 6, h(t) = 15

[tex]Speed = \frac{15}{6} = 2.5[/tex]

At t = 9, h(t) = 11

[tex]Speed = \frac{11}{9} = 1.2[/tex]

At t = 12, h(t) = 6

[tex]Speed = \frac{6}{12} = 0.5[/tex]

At t = 15, h(t) = 0

[tex]Speed = \frac{0}{15} = 0[/tex]

By comparing the above values, we notice that as time and height increases, the value of speed reduces.

This means that the greatest value of speed will be at the least value of time (t).

From the given options, the least value of time is at:

[tex]t = 2[/tex]

Hence, the value of time (t) at the greatest speed of the ball is at 2 seconds

Read more about speed at:

https://brainly.com/question/7359669

Select the statement that best justifies the conclusion based on the given information.

l is in plane M,
x is on line l
Conclusion: x is in plane M.

a. A plane contains at least three points not all on the same line.
b. If two points lie in a plane, then the line containing them lies in that plane.
c. If a plane contains a line, it contains the points on the line.
d. Exactly one plane contains a given line and a point not on the line.

Answers

9514 1404 393

Answer:

  c. If a plane contains a line, it contains the points on the line.

Step-by-step explanation:

The only statement relating a point on a line to the plane containing the line is the one shown above.

_____

Additional comment

Identifying true statements is a reasonable strategy for many multiple-choice questions. Another strategy that can be employed is finding the one true statement that is relevant to the question being asked.

find the sum or difference of 4/5 - (-3 4/5)

Answers

Answer:

4 3/5

Step-by-step explanation:

4/5 - (-3 4/5)

Subtracting a negative is like adding

4/5 + 3 4/5

3 8/5

3 5/5 + 3/5

3+1+3/5

4 3/5

find the product
(4\m+m)(4/m-m)

Answers

[tex]\\ \sf\longmapsto \dfrac{4}{m+m}\times \dfrac{4}{m-m}[/tex]

[tex]\\ \sf\longmapsto \dfrac{4\times 4}{(m+m)(m-m)}[/tex]

[tex]\boxed{\sf (a-b)(a+b)=a^2-b^2}[/tex]

[tex]\\ \sf\longmapsto \dfrac{16}{m^2-m^2}[/tex]

[tex]\\ \sf\longmapsto \dfrac{16}{0}[/tex]

[tex]\\ \sf\longmapsto \infty[/tex]

Answer:

(16-m^4)/m^2

Step-by-step explanation:

=([tex]\frac{4}{m}[/tex]+m)([tex]\frac{4}{m}[/tex]-m)

=[tex]\frac{4+m^2}{m}[/tex]*[tex]\frac{4-m^2}{m}[/tex]  (LCM)

[tex]\frac{16-m^4}{m^2}[/tex] (a-b)(a+b)

If the sequence 2, 4, 6, 10, 16, ... were to follow the same pattern as the Fibonacci sequence, what are the next three terms?

Answers

Answer:

26, 42, 68

Step-by-step explanation:

In the Fibonacci sequence, each number is equal to the sum of the two previous numbers. Denoting the sequence as a, we can say that

a₁=2

a₂=4

a₃=6

a₄=10

a₅=16

What we can see here is that each number a₃ or above is equal to the sum of its two previous numbers. For example, a₃ = a₁+a₂ = 2 +4 = 6

Therefore,

a₆ = a₄+a₅ = 10+16 = 26

a₇ = a₅+a₆ = 16 + 26 = 42

a₈ = a₆+a₇ = 26 + 42 = 68

Suppose the discrete random variable X has the probability distribution below:

X 0 1 2 3 4
P(X) 0.11 0.52 0.19 0.12 0.06
1pt a right parenthesis space F i n d space P left parenthesis X less than 3 right parenthesis

2pt b right parenthesis space F i n d space P left parenthesis X greater or equal than 1 right parenthesis

2pt c right parenthesis space F i n d space mu subscript X

2pt, 1pt d right parenthesis space F i n d space sigma subscript X squared space a n d space sigma subscript X

Answers

(a) P(X < 3) = P(X = 0) + P(X = 1) + P(X = 2) = 0.11 + 0.52 + 0.19 = 0.82

(b) P(X ≥ 1) = P(X = 1) + P(X = 2) + P(X = 3) + P(X = 4) = 0.52 + 0.19 + 0.12 + 0.06 = 0.89

(c) µ = 0×0.11 + 1×0.52 + 2×0.19 + 3×0.12 + 4×0.06 = 1.5

(d) σ² = (0²×0.11 + 1²×0.52 + 2²×0.19 + 3²×0.12 + 4²×0.06) - µ² = 1.07

σ = √(σ²) ≈ 1.03

The midterm exam will have a total of 108 marks and there are twice as many 05 marks questions than 02 marks questions. Each question is worth either 02 marks or 05 marks”. Find out how many questions of each value there are in the online test?​

Answers

Answer: 9 (2 mark question) and 18 (5 marks questions)

Step-by-step explanation:

Given: Total mark = 108

Let the 2 marks questions be x

Therefore 5 mark question = 2x

ATQ

⇒5(2x)+2(x) = 108

⇒10x+2x =108

⇒12x = 108

⇒x = 108/12

⇒x = 9

Total two mark questions are 9 and 5 mark questions are 18.

please click thanks and mark brainliest if you like :)

how do you tell how many times greater is the 5 in 458,039 than the 5 in 271,145

Answers

Answer:

count the number of places it is. The 5 in 458,039 is in the 10,000 place and the 5 in 271,145 is in the ones place so the 5 in 458,039 is 10,000 times greater

1,000
the 5 in 45,039 has a value of 1,000 while the 5 in 271,145 has a value of 1 and 1,000/1=1,000

Match the expressions given in words with their algebraic expressions.
&+59
5(5—9)
5+39
5(9-5)
-56

Answers

Answer:

5+39 this is the answer 5+39

HELP! I really need the answer quick!!!!!!!

Answers

Answer:

139°

Step-by-step explanation:

8x+51+6x-25=180

14x+26=180

14x=180-26

14x=154

x=154/14

x=11

<AOB= 8x+51 = 8(11)+51 = 88+51 = 139

Why school sucks: A makes you learn stuff you don’t care about and will never use B teachers suck and don’t teach property C they don’t teach you useful things like taxes bills ect D they overwork students E its not even about learning anymore all goals are is just good grades.

Why is our future determined on letters on a piece of paper? I’d rather die then be in this society

Answers

Answer:

I think answer is E

Step-by-step explanation:

Answer Fasho F no kapp all facts

a² +6²
a-b
if a = 3 and b = 4
Evaluate each expression using the variable replacements.

Answers

Knowing what the variables are means that all we have to do is substitute:
[ ( 3 ) ^2 + ( 4 ) ^2 ] / 4 - 3 =
( 9 + 16 ) / 1 =
25

Answer:

-45

Step-by-step explanation:

let a= 3 and b= 4a² + 6² / a - b= 3² + 6² / 3 - 4= 9 + 36 / -1= 45 / -1= -45

[tex]\tt{ \green{P} \orange{s} \red{y} \blue{x} \pink{c} \purple{h} \green{i} e}[/tex]

Divide and check by multiplying the quotation by the divisor 8m^4+12m^3 over 4m

Answers

Answer:

2m^3 + 3m^2

Step-by-step explanation:

8m^4+12m^3

---------------------

4m

2m^3 + 3m^2

Check

4m(2m^3+3m^2)

8m^4 + 12 m^3

A sample tested the claim that heights of men and heights of women have difference​ variances, with s=7.42388 cm for women and 7.14974 cm for men. The sample sizes are n1=144 and n2=156. When using the F test with these​ data, is it correct to reason that there is no need to check for normality because n1>30 and n2>​30?

Answers

No. The F test has a requirement that samples be from the normally distributed populations, regardless of how large the samples are.

The F-test simply shows whether the variances that are in the numerator and the denominator are equal. The F-test can be applied on a large sampled population.

One main assumption of the F test is that the populations where the two samples are drawn are normally distributed.

Regarding the question, it's important to note that when using the F test with these​ data, it's not correct to reason that there is no need to check for "normality".

It should be noted that the F test has a requirement that samples are from the normally distributed populations, regardless of how large such samples are.

Read related link on:

https://brainly.com/question/16786843

7. Kylie bikes at a speed of 100 yards per minute. Robert bikes at a speed of 240 feet per minute. In feet per second, how much faster does Kylie bike than Robert?​

Answers

440 feet she get equal

a display order of numbers are called​

Answers

Answer:

I think

A display order of numbers are called​ sequences.

The above answer is correct


Which expression is the best estimate of the product of 7/8and 8 1/10?

Answers

Answer:

7 7/80 or 7.0875

Step-by-step explanation:

product is the result of multiplication

7/8 * 81/10 = 567/80 = 7 7/80 or 7.0875

Other Questions
IN YOUR OWN WORDS, what are the definitions for Interkinesis (include when it happens)? What is the true solution to the equation below? 2 lne^ln2x-lne^ln10x=ln30 If anyone knows the answer plz tell me, thank you Need help! Need help! Need help! Need help! Need help! Need help! 125. Albert surveyed a class of 25 students on sports. 5 kids love baseball. 7 kids love basketball. 10 kidslove football. How many students did not like baseball, basketball, or football?25 students12 students22 students3 students An electron and a proton are initially very far apart (effectively an infinite distance apart). They are then brought together to form a hydrogen atom, in which the electron orbits the proton at an average distance of 5.29x10-11m. What is the change in electric potential energy? The tabulated data show the rate constant of a reaction measured at several different temperatures. Use an Arrhenius plot to determine the activation barrier and frequency factor for the reaction.Temperature (K) Rate Constant (1/s)300 0.0134310 0.0407320 0.114330 0.303340 0.757Required: Use an Arrhenius plot to determine the activation barrier for the reaction. Summarise the following sentence in not more than ten (10) words:Miss Brill is the story told brilliantly and realistically, balancing thoughts and emotions that sustain the late solitary life amidst all the bustle of modern life of an old woman Given the following constraints, find the maximum and minimum values for z. Constraints: 2xy124x+2y0x+2y6 Optimization Equation: z=2x+5y World War II was responsible for inspiringA. colonial independence movements.B. an end to communism.C.a new wave of imperialism.D. unequal treaties. fastest answer gets brainiest !Which data value has the highest frequency?1163163858 Can someone please help me solve this problem Choose the best answer to complete the sentences based on the reading in Cultura. Questions is Barcelona's soccer stadium. In Spain, the between the two main soccer teams has always existed. A game between Real Madrid and is always a great social, cultural, and even political event. The Colombian team is the famous rival of the team Millonarios. Two soccer teams a game when they score the same number of goals. To play soccer you need a . The person who referees a soccer game is the . A worldwide championship is a . HELPWhich of the following does NOT incorporate phosphorus?A. PhospholipidsB. Nucleotides. D. Carbohydrates Choose the correct relative pronoun.Todo you pay your rent?O whosewhomwhowhat NEED HELP ASAP I HAVE 3 MINS i Needdd helppp with this?? Job analysis is important to HR managers because the information gathered in job analysis is used in so many HR activities/functions. Describe how job analysis information is used in four different HR activities/functions. Illustrate by specific examples in hospitality and tourism. 9 3/5 % as a decimal, rounded to 3 decimal places, is: